Reduce fatalities in Airline Collisions

This topic has expert replies
Master | Next Rank: 500 Posts
Posts: 201
Joined: Sat Jul 10, 2010 2:23 pm
Thanked: 7 times
Followed by:1 members

Reduce fatalities in Airline Collisions

by ov25 » Sat Oct 23, 2010 6:56 am
The number of aircraft collisions on the ground is increasing because of the substantial increase in the number of flights operated by the airlines. Many of the fatalities that occur in such collisions are caused not by the collision itself, but by an inherent flaw in the cabin design of most aircraft, in which seats, by restricting access to emergency exits, impede escape. Therefore, to reduce the total number of fatalities that result annually from such collisions, the airlines should be required to remove all seats that restrict access to emergency exits.

Which one of the following proposals, if implemented together with the proposal made in the passage, would improve the prospects for achieving the stated objective of reducing fatalities?
(A) The airlines should be required, when buying new planes, to buy only planes with unrestricted access to emergency exits.
(B) The airlines should not be permitted to increase further the number of flights in order to offset the decrease in the number of seats on each aircraft.
(C) Airport authorities should be required to streamline their passenger check-in procedures to accommodate the increased number of passengers served by the airlines.
(D) Airport authorities should be required to refine security precautions by making them less conspicuous without making them less effective.
(E) The airlines should not be allowed to increase the ticket price for each passenger to offset the decrease in the number of seats on each aircraft.

User avatar
Master | Next Rank: 500 Posts
Posts: 108
Joined: Fri Jul 09, 2010 8:15 am
Location: 127.0.0.1
Thanked: 15 times

by gmatrix » Sat Oct 23, 2010 7:20 am
hmmm....my pick B

Which one of the following proposals, if implemented together with the proposal made in the passage, would improve the prospects for achieving the stated objective of reducing fatalities?
two reasons for number of aircraft collisions- more no. of flights and blocked exits The latter one i.e.remove restriction to exits is already mentioned in the conclusion-Therefore, to reduce the total number of fatalities that result annually from such collisions, the airlines should be required to remove all seats that restrict access to emergency exits

the other is- curtailing the increase of number of flights is to be addresed....as the airlines could try and increase the no.of flights to make up for the lost profits incurred by the removal of seats thereby increasing the no. of flights and risking more collisions ......hence B fits the bill
Life is all about ass; you're either covering it, laughing it off, kicking it, kissing it, busting it, trying to get a piece of it, or behaving like one.

User avatar
GMAT Instructor
Posts: 23
Joined: Sun Sep 12, 2010 1:40 am
Thanked: 10 times
Followed by:7 members

by Amit@EconomistGMAT » Sun Oct 24, 2010 2:57 am
Interesting question, as it is actually an Argument Flaw or Assumption question in disguise. We are asked to locate the flaw in the argument's plan and solve it with one of the suggested plans.

However, it is more likely that the answer is A. The problem with B is that it is designed to prevent the number of flights from rising and thereby prevent the number of collisions from rising. However, we are explicitly told that the goal is reducing fatalities, and that fatalities are caused not by collisions, but by the design flaw. Answer choice A completes the partial coverage against the design flaw that is suggested by the argument, and is therefore the correct one.
Amit Moshe
Verbal Section Instructor
Economist GMAT

User avatar
Master | Next Rank: 500 Posts
Posts: 286
Joined: Tue Sep 21, 2010 5:36 pm
Location: Kolkata, India
Thanked: 11 times
Followed by:5 members

by pesfunk » Sun Oct 24, 2010 7:16 am
IMO A ;)

OA Please ?
Amit Moshe wrote:Interesting question, as it is actually an Argument Flaw or Assumption question in disguise. We are asked to locate the flaw in the argument's plan and solve it with one of the suggested plans.

However, it is more likely that the answer is A. The problem with B is that it is designed to prevent the number of flights from rising and thereby prevent the number of collisions from rising. However, we are explicitly told that the goal is reducing fatalities, and that fatalities are caused not by collisions, but by the design flaw. Answer choice A completes the partial coverage against the design flaw that is suggested by the argument, and is therefore the correct one.

Master | Next Rank: 500 Posts
Posts: 201
Joined: Sat Jul 10, 2010 2:23 pm
Thanked: 7 times
Followed by:1 members

by ov25 » Sun Oct 24, 2010 6:23 pm
I like Amit's explanation...however, the OA is not A.

I actually picked A myself but on a second thought it appears that A is a repetition -- the airlines should be required to remove all seats that restrict access to emergency exits.

It appears as though the question is a strengthen type question and is interested the answer that, in addition to the quoted, helps with fatalities in such collisions -- collisions that cause more fatalities due to stampede at the emergency exit.

I guess I should wait for more discussion before posting OA.

User avatar
Senior | Next Rank: 100 Posts
Posts: 44
Joined: Tue Jun 22, 2010 12:15 am
Thanked: 3 times

by showbiz » Sun Oct 24, 2010 6:47 pm
I think the answer is B. Although I have seen a trend that in CR, the obvious answer (something that seems to already exist in the passage) is the right answer...in this case A.

My answer is B, but wouldn't be surprised if its A.

OA pls?

Junior | Next Rank: 30 Posts
Posts: 17
Joined: Wed Sep 29, 2010 8:14 am
Thanked: 1 times

by vijchid » Mon Oct 25, 2010 1:06 pm
IMO B. A seems to repeat what is already given. The first line in the passage says that the number of collisions are increasing. To decrease the fatalities they are already doing something about the seats and we need to do something to reduce the collisions(which in turn will reduce the fatalities).

User avatar
Master | Next Rank: 500 Posts
Posts: 154
Joined: Thu Aug 26, 2010 9:32 am
Location: Chicago,IL
Thanked: 46 times
Followed by:19 members
GMAT Score:760

by rkanthilal » Mon Oct 25, 2010 1:46 pm
IMO B. I choose B because the argument states that many fatalities (rather than all fatalities) are caused by the seats restricting access to the exits. And the conclusion states that in order to reduce (not eliminate) the number of fatalities airlines should be required to remove the seats that restrict access to emergency exits. So it is possible that even if all airplanes (old and new) have seats that do not restrict the exits, fatalities can still go up if the number of accidents goes up substantially. In other words some fatalities are caused by the seats and some are caused by other reasons. The conclusion addresses the fatalities that are caused by the seats. We need to provide a solution that complements the conclusion and restricts the fatalities from other causes from going up. Answer B does this. The argument states that the number of collisions is increasing because of the increase in traffic. If the traffic stays the same then the number of collisions will stay the same. If the number of collisions stays the same, then the number of fatalities from other causes will stay the same. This combined with the decrease in the number of fatalities from the seats will lead to a decrease in the total fatalities.

User avatar
Master | Next Rank: 500 Posts
Posts: 154
Joined: Thu Aug 26, 2010 9:32 am
Location: Chicago,IL
Thanked: 46 times
Followed by:19 members
GMAT Score:760

by rkanthilal » Mon Oct 25, 2010 1:51 pm
Additionally, with regard to answer choice A, the conclusion states that "the airlines should be required to remove all seats that restrict access to emergency exits". So if an airline buys a new plane with seats that restrict the exits, wouldn't they be required to immediately remove those seats before putting the plane in operation. Choice A seems redundant.

GMAT Instructor
Posts: 1302
Joined: Mon Oct 19, 2009 2:13 pm
Location: Toronto
Thanked: 539 times
Followed by:164 members
GMAT Score:800

by Testluv » Mon Oct 25, 2010 10:44 pm
received a pm.

Choice B is definitely correct.

This question is asking you to add a proposal to the one discussed in the stimulus such that the problem (ie, fatalities) would be diminished. In problem/plan arguments, be sure to understand every aspect of the problem (as well as the plan).

What's causing the problem of fatalities here?

Well, if you look at the very first line of the argument you see the evidence keyword "because." So, in the first sentence we learn one reason why there are more collisions: there are more flights. In the second sentence, you learn another reason: the cabin design flaw that blocks emergency exit.

So, the best way to fix this is to target these two sources of the problem: a) fix the cabin design flaw and b) ensure that the number of flights does not increase. That's why choice B is correct.

***

Choice A is wrong because it doesn't actually ADD anything. (We already learned that they are removing the seats that block the exits. Thus, choice A is redundant; because it's redundant it can't improve the prospects of reducing fatalities).
Kaplan Teacher in Toronto

User avatar
GMAT Instructor
Posts: 2193
Joined: Mon Feb 22, 2010 6:30 pm
Location: Vermont and Boston, MA
Thanked: 1186 times
Followed by:512 members
GMAT Score:770

by David@VeritasPrep » Wed Jan 12, 2011 10:57 am
OA is B.

I received a PM asking me to resolve the conflict between the many earlier postings on this question, judging by the dates of the earlier postings we can such much earlier - back in October! I found that there was not much of a conflict judging from the great postings on this one by rkanthilal, TestLuv (expert), vijchid, showbiz, and OV25. Rather than just restate the great explanations they have given I am going to turn this into "How to approach an Inference Plan question"

First a little more information on this one: It must be LSAT day on Beat the GMAT. This is the third question I have been asked about today without a source that turns out to be an official LSAT question. Most of you are using LSAT questions to study, even if you do not know it!

I found that this question is from the October 1993 LSAT test, it is from the First Logical Reasoning Section, question 10. Interestingly, this is one of the questions where two questions come from one stimulus. So question #9 is also from this stimulus. I will post that question in this thread in case people who have tried the above question would like to do #9 as well.

The official answer to this question is B.

This a difficult variation on a type of question that you would see on the GMAT - a PLAN question. (Please see other postings I have made on plan questions at the following links: https://www.beatthegmat.com/businesses-t ... tml#313911 https://www.beatthegmat.com/strenghtenin ... tml#313159).

How do you identify the Plan question? You can tell based on the question stem. A plan question might say "plan" or "proposal" as this one does - and if so it is a plan!

But a great way to recognize a plan question is that it has a goal mentioned in the question stem - while other questions do not. This particular question says, "Which one of the following proposals...would improve the prospects for achieving the stated objective of reducing fatalities?" So there is the goal, the objective of reducing fatalities. So we know that this is a plan question.

As I have mentioned in the postings listed above there are three kinds of plan questions:

1) The strengthen a plan question

2) The weaken a plan question

3) The inference plan question.

This question is an inference plan question. The question stem says, ""Which one of the following proposals...would improve the prospects for achieving the stated objective..." So the question stem indicates that you are selecting the best plan (answer choice) to achieve the stated goal.

When you read the stimulus and question stem you need to focus on the goal that you are trying to achieve and the background information that will help you to understand the situation and how this relates to the plan the goal.

As TestLuv indicated in an earlier posting, in this particular problem we have two causes of the fatalities. The first cause is the fact that there are collisions on the ground at the airports. The stimulus makes it clear that the cause of these collisions is "the substantial increase in the number of flights operated by the airlines."

The second cause of the fatalities, and the one that the test writers try to get you to focus on is the "inherent flaw in the cabin design of most aircraft, in which seats, by restricting access to emergency exits, impede escape." This is the problem that the first proposal, the one in the stimulus addresses. The proposal in the stimulus is to remove the seats blocking the emergency exits. This would seem to address the second problem. However, the test writers have you focused on this problem so when you head to the answer choices you may be only thinking of this proposal.

Moving to the answer choices, Choice C, D, and E all address peripheral issues such as the cost of tickets, the security procedures, and the check-in procedures. This leaves choices A and B, which are the two that have been debated above.

Answer Choice A focuses on the plan that has already been stated in the stimulus above: the plan to improve access to the emergency exits. As stated in earlier posts above this does not actually bring us new information or even a new proposal.

Answer Choice B focuses on the other part of the problem that has not been addressed, the increased number of flights. Without the plan in choice B what would happen is possibly a tradeoff, better access to the emergency exits, but if there are more flights there may be more collisions. This was pointed out very nicely above by rkanthilal.

Hope it helps!
Veritas Prep | GMAT Instructor

Veritas Prep Reviews
Save $100 off any live Veritas Prep GMAT Course

User avatar
GMAT Instructor
Posts: 2193
Joined: Mon Feb 22, 2010 6:30 pm
Location: Vermont and Boston, MA
Thanked: 1186 times
Followed by:512 members
GMAT Score:770

by David@VeritasPrep » Wed Jan 12, 2011 11:16 am
I just posted the other question from the same stimulus (as I mentioned above) at the following link:

https://www.beatthegmat.com/lsat-strengt ... tml#330803
Veritas Prep | GMAT Instructor

Veritas Prep Reviews
Save $100 off any live Veritas Prep GMAT Course

User avatar
Master | Next Rank: 500 Posts
Posts: 439
Joined: Sat Aug 09, 2008 8:32 am
Location: India
Thanked: 34 times
Followed by:28 members

by sivaelectric » Fri May 27, 2011 1:03 am
My Choice is also B
If I am wrong correct me :), If my post helped let me know by clicking the Thanks button ;).

Chitra Sivasankar Arunagiri

Master | Next Rank: 500 Posts
Posts: 160
Joined: Tue Jul 07, 2009 1:09 pm
Thanked: 1 times
Followed by:1 members

by Sharma_Gaurav » Tue Apr 03, 2012 3:04 am
coice B is correct
totally agree with testluv reasoning, I also employed the same reasoning to arrive at B

Legendary Member
Posts: 1404
Joined: Tue May 20, 2008 6:55 pm
Thanked: 18 times
Followed by:2 members

by tanviet » Tue Apr 03, 2012 8:06 pm
David@VeritasPrep wrote:OA is B.

I received a PM asking me to resolve the conflict between the many earlier postings on this question, judging by the dates of the earlier postings we can such much earlier - back in October! I found that there was not much of a conflict judging from the great postings on this one by rkanthilal, TestLuv (expert), vijchid, showbiz, and OV25. Rather than just restate the great explanations they have given I am going to turn this into "How to approach an Inference Plan question"

First a little more information on this one: It must be LSAT day on Beat the GMAT. This is the third question I have been asked about today without a source that turns out to be an official LSAT question. Most of you are using LSAT questions to study, even if you do not know it!

I found that this question is from the October 1993 LSAT test, it is from the First Logical Reasoning Section, question 10. Interestingly, this is one of the questions where two questions come from one stimulus. So question #9 is also from this stimulus. I will post that question in this thread in case people who have tried the above question would like to do #9 as well.

The official answer to this question is B.

This a difficult variation on a type of question that you would see on the GMAT - a PLAN question. (Please see other postings I have made on plan questions at the following links: https://www.beatthegmat.com/businesses-t ... tml#313911 https://www.beatthegmat.com/strenghtenin ... tml#313159).

How do you identify the Plan question? You can tell based on the question stem. A plan question might say "plan" or "proposal" as this one does - and if so it is a plan!

But a great way to recognize a plan question is that it has a goal mentioned in the question stem - while other questions do not. This particular question says, "Which one of the following proposals...would improve the prospects for achieving the stated objective of reducing fatalities?" So there is the goal, the objective of reducing fatalities. So we know that this is a plan question.

As I have mentioned in the postings listed above there are three kinds of plan questions:

1) The strengthen a plan question

2) The weaken a plan question

3) The inference plan question.

This question is an inference plan question. The question stem says, ""Which one of the following proposals...would improve the prospects for achieving the stated objective..." So the question stem indicates that you are selecting the best plan (answer choice) to achieve the stated goal.

When you read the stimulus and question stem you need to focus on the goal that you are trying to achieve and the background information that will help you to understand the situation and how this relates to the plan the goal.

As TestLuv indicated in an earlier posting, in this particular problem we have two causes of the fatalities. The first cause is the fact that there are collisions on the ground at the airports. The stimulus makes it clear that the cause of these collisions is "the substantial increase in the number of flights operated by the airlines."

The second cause of the fatalities, and the one that the test writers try to get you to focus on is the "inherent flaw in the cabin design of most aircraft, in which seats, by restricting access to emergency exits, impede escape." This is the problem that the first proposal, the one in the stimulus addresses. The proposal in the stimulus is to remove the seats blocking the emergency exits. This would seem to address the second problem. However, the test writers have you focused on this problem so when you head to the answer choices you may be only thinking of this proposal.

Moving to the answer choices, Choice C, D, and E all address peripheral issues such as the cost of tickets, the security procedures, and the check-in procedures. This leaves choices A and B, which are the two that have been debated above.

Answer Choice A focuses on the plan that has already been stated in the stimulus above: the plan to improve access to the emergency exits. As stated in earlier posts above this does not actually bring us new information or even a new proposal.

Answer Choice B focuses on the other part of the problem that has not been addressed, the increased number of flights. Without the plan in choice B what would happen is possibly a tradeoff, better access to the emergency exits, but if there are more flights there may be more collisions. This was pointed out very nicely above by rkanthilal.

Hope it helps!
Thank you expert for great posting.

I want you to help me in general problem.
I heard that there are 2 way of strengthening/weakening an argument. The first way is to validate/invalidate an assumption of the argument. The second way is to support/weaken the conclusion directly. I already understand the first way. Most CR questions in OG books belong to the first way. I do not understand the second way. Pls, give an example of the second way, in which a strengthener/weakener support/weaken the conclusion directly. Thank you.